Warum teilen sich Photonen nicht in mehrere niedrigere Energieversionen ihrer selbst auf?

Ein Photon könnte sich spontan in zwei oder mehr Versionen seiner selbst aufteilen, und alle mir bekannten Erhaltungssätze würden durch diesen Prozess nicht verletzt. (Glaube ich.) Ich habe darüber nachgedacht, und ein System, das aus mehreren Photonen mit niedrigerer Energie besteht, hätte eine signifikant höhere Anzahl von Mikrozuständen (und folglich eine höhere Entropie) als eines, das aus einem einzelnen Photon mit so viel Energie besteht. Dies würde das Verfahren günstiger machen.

Warum passiert das nicht?

Kannst du eine machen γ 2 γ Vertex (Wechselwirkungsterm im Lagrange) der eichinvariant ist?
Nein (weil ich noch nicht so viel gelernt habe!) Könntest du es vielleicht einfacher erklären?
Um in Feynman-Diagrammen auf Baumebene zu sein, müssen Sie einen Begriff in der Lagrange-Funktion haben EIN 3 zum γ 2 γ , wo EIN ist das Photonenvektorfeld. Ich weiß nicht, wie ich dies entweder Lorentz- oder Eich-invariant machen soll, die beide grundlegende Symmetrien der Natur sind. Beachten Sie, dass dies nicht bedeutet, dass dieser Zerfall nicht über Diagramme höherer Ordnung erfolgen kann, weshalb dies keine Antwort ist :)
Ein Photon kann sich nach Wechselwirkung mit einer Ladung aufspalten, aber dieser Vorgang hat eine sehr kleine Amplitude. Es ist eine vorgeschlagene Übung im QFT-Buch von Itzykson und Zuber im Abschnitt über Photon-Photon-Streuung.
Vorherige Version dieser Frage: physical.stackexchange.com/questions/12488/…
Ein einzelnes Photon hat einen Impuls in eine ganz bestimmte Richtung. Nehmen Sie für die Argumentation an, dass er sich entlang der positiven x-Achse in positiver Richtung bewegt. Damit sich dieses Photon in zwei Teile aufspaltet, müsste es einen kleinen Impuls in vertikaler (oder Tiefe – „z“) Richtung geben. Wenn dieser vertikale Impuls plötzlich aus dem Nichts auftaucht, hätten Sie die Impulserhaltung verletzt, da der Gesamtimpuls UND jede Komponente des Gesamtimpulses in jeder Wechselwirkung erhalten bleibt. Das bedeutet, dass der erste Satz in Ihrer Problemstellung wahrscheinlich falsch ist.
Gute Frage. Zum Zeitpunkt, an dem ich dies schreibe, gibt es zwei beliebte Antworten, die zusammen mehr als 21 Stimmen erhalten. Aber beides ergibt für mich keinen Sinn. Entweder liegen beide falsch, oder sie sind beide schlecht dargestellt.

Antworten (7)

Nach der hypothetischen Teilung würden sich 2 Photonen mit der gleichen Energie unter einem Winkel ausbreiten, der bei Impulserhaltung ok ist. Dann gäbe es ein Ruhesystem, bei dem der Winkel 180 Grad beträgt. Wenn Sie nun in diesem Ruherahmen bleiben und vor der Teilung in die Zeit zurückgehen, wäre Ihr einzelnes Photon in Ruhe. Das ist jedoch nicht möglich: Gemäß der Relativitätstheorie ist die Lichtgeschwindigkeit für alle Frames konstant. Somit kann im Vakuum (dh ohne Impulsübertragung während der Aufspaltung) ein einzelnes Photon nicht in zwei aufgespalten werden. Mathematisch liegt der Grund darin, dass die Lorentz-Gruppe nicht kompakt ist, was bedeutet, dass der Parameter Gamma jeden Wert von [1, unendlich) annehmen kann, aber nicht unendlich selbst, was einem Koordinatenrahmen entsprechen würde, der sich mit Lichtgeschwindigkeit bewegt, wobei alle massiven Teilchen unendlich sind kinetische Energie.

Dies scheint ein Zirkelschluss zu sein: Untersuchen Sie den physikalischen Prozess X in einem Rahmen, der gemäß der speziellen Relativitätstheorie nicht existiert. Da der Rahmen nicht existieren kann, muss der Prozess dann unmöglich sein. Anders ausgedrückt könnte man genauso gut sagen, dass ein einzelnes Photon vor der Aufspaltung nicht existieren kann. Denn in irgendeinem Rahmen muss es ruhen, aber ein solcher Rahmen ist unmöglich. Kreisförmig.
rdjain1, das Argument ist weder zirkulär noch auf andere physikalische Prozesse verallgemeinerbar. Tatsächlich geht das Argument nicht in einen verbotenen Rahmen an sich, sondern in den Ruherahmen. Nun ist der einzige physikalische Prozess, der kein Ruhesystem hat, das einzelne Photon, das sich im Vakuum ausbreitet. Diese Tatsache führt dazu, dass sich einzelne Photonen nicht aufspalten können, obwohl es ihnen vielleicht erlaubt wäre, wenn man nur Energie- und Impulserhaltung betrachtet.
Diese Erklärung ist falsch: Zwei kollineare Photonen mit gleicher Energie haben einfach kein Ruhesystem: Das Ruhesystem für das Photonenpaar ist das gleiche wie für das Ruhesystem eines einzelnen Photons. Aus rein kinematischer Sicht ist es völlig erlaubt, dass ein Photon zu zwei Photonen in die gleiche Richtung mit der gleichen Energie (der Hälfte der Summe) geht.
@Chris Da stimme ich voll und ganz zu. Ein N -fach aufgeteilt in N Impulszustände kollinear mit dem Original (und mit dem gleichen Sinn) und jeweils mit N 1 mal der ursprünglichen Energie / 3-Impuls ist die eindeutige Lösung, die mit der Erhaltung des 4-Impulses vereinbar ist. In keinem Fall gibt es einen Massenschwerpunktrahmen. Zu all dem gehört definitiv noch mehr.
@WetSavannaAnimalakaRodVance Es kommt darauf an, dass es kinematisch kaum erlaubt ist - es gibt nur einen Punkt im Phasenraum der Tochterteilchen, der dies zulässt. Die Abklingrate beinhaltet ein Integral über den Phasenraum, und da der Integrand nur an einem Punkt nicht null (und endlich) ist, ergibt das Integral null.
Diese Antwort ist völlig falsch, und es ist eine Schande, dass dies die Top-Antwort auf eine so wichtige Frage ist!
Diese Antwort ist immer noch falsch.
Es scheint mir, dass diese Antwort "fast richtig" ist, mit Ausnahme des Punktes, der in Chris 'Kommentar oben angesprochen wurde. Der verbleibende kolineare Fall wurde in knzhous Antwort auf dieselbe Frage hier behandelt .

Ein Photon ist ein Elementarteilchen. So viel Elementar und so viel Teilchen wie das Elektron.

Ein einzelnes Elementarteilchen hat eine feste Masse und kann kein anderes Teilchen emittieren, ohne die Energieerhaltung zu verletzen, weil seine Masse fest ist. Im Massenschwerpunkt eines massiven Elementarteilchens, Elektron, gibt es keine Energie für eine Emission, für ein strahlendes Elektron in einem Feld wird die Energie vom Feld geliefert.

Wenn ein masseloses Elementarteilchen wie das Photon in zwei Teile geteilt werden könnte, erscheint plötzlich eine invariante Masse und das vor der Teilung null invariante Masse, nach der Teilung eine messbare invariante Masse. Dies bedeutet, dass sowohl die Impuls- als auch die Energieerhaltung verletzt werden , da die invariante Masse das Maß des Vierervektors vor und nach der Teilung ist. Ein Photon kann auch in Diagrammen höherer Ordnung mit einem Feld interagieren, aber nicht in dem Sinne spalten, wie Sie es sich vorstellen.

Edit nach Diskussion in den Kommentaren:

Angenommen, ein Photon könnte in zwei Photonen zerfallen.

Diese Photonen haben vier Vektoren. Es gibt zwei Situationen: Ihre drei Impulse sind im Labor parallel zum ursprünglichen Photon, oder es gibt einen Winkel der drei Impulse mit dem ursprünglichen Photon und auch zwischen ihnen. Im letzteren Fall definieren die beiden Zerfallsphotonen einen Massenschwerpunkt (ähnlich einem ruhenden Pi0). In diesem System addieren sich die beiden Impulse zu Null, aber es wird Energie geben, die dem System eine unveränderliche Masse verleiht, was gegen die Energieerhaltung verstößt, da das ursprüngliche Photon eine unveränderliche Masse von 0 hatte, dh diese Energie nicht liefern kann. Das ursprüngliche Photon im Massenschwerpunkt der zerfallenen Photonen bewegt sich immer noch mit der Geschwindigkeit c und hat daher einen von Null verschiedenen Impuls, sodass auch die Impulserhaltung verletzt wird.

Im Fall von zwei kollinearen Photonen im Labor wird ihre unveränderliche Masse Null sein, wenn der Grenzwinkel zwischen ihnen genau 0 ist, andernfalls gilt das obige Argument. Wenn es genau 0 ist, kann kein Massenmittelpunkt definiert werden, da sich ein masseloses System mit Lichtgeschwindigkeit bewegt.

Es stellt sich also die Frage: Warum verwandelt sich ein Photon der Frequenz nu im Labor nicht in zwei exakt kollineare Photonen niedrigerer Frequenz. Experimentell wurde dies nicht beobachtet. Wenn es also passieren kann, ist es ein Prozess mit sehr sehr geringer Wahrscheinlichkeit. In den Kommentaren gibt Lubos Motl diese Aussage ab: "Für Photonen ist diese Amplitude aufgrund der Abelschen Eichsymmetrie und anderer Symmetrien 0." Ich suche noch einen Link dazu.

In der nächsten Antwort wird der kollineare Fall durch die spezielle Relativitätstheorie ausgeschlossen,

Mathematisch liegt der Grund darin, dass die Lorentz-Gruppe nicht kompakt ist, was bedeutet, dass der Parameter Gamma jeden Wert von [1, unendlich) annehmen kann, aber nicht unendlich selbst, was einem Koordinatenrahmen entsprechen würde, der sich mit Lichtgeschwindigkeit bewegt, wobei alle massiven Teilchen unendlich sind kinetische Energie.

Photonen haben Chiralität, daher sollten Sie auch die Drehimpulserhaltung berücksichtigen. Zum 1 γ 2 γ Streuung, dies ist nicht möglich. (Ich gehe davon aus, dass nur kollineare Photonen erzeugt werden. Es ist offensichtlich, dass Energie- und Impulserhaltung verletzt werden, wenn zwei nicht kollinear sind.)

Ich bin kein Teilchenphysiker, also werde ich fragen, warum das anfängliche Photon nicht linear polarisiert werden konnte, dh in einer Überlagerung von zwei entgegengesetzten AM-Zirkularpolarisationen, so dass der erwartete Wert des beobachtbaren Spins Null war?
Genauer gesagt sei der Zustand der beiden Photonen: $$\alpha c^{\dagger } _{p \uparrow}c^{\dagger } _{p \uparrow}
Entschuldigung, mein vorheriger Kommentar war falsch und ich kann den obigen nicht löschen, ich werde eine Antwort geben.

So sieht die Reaktion aus γ 2 γ ist nicht nur dynamisch verboten (Theorem von Furry), sondern auch kinematisch verboten.

Wie Dexter Kim betont, besteht die einzige Möglichkeit, Energie und Impuls zu erhalten, darin, dass die beiden Photonen bei emittiert werden 0 ° , wobei der Drehimpuls entlang der Bewegungsrichtung durch die Kopplung der beiden Photonenspins gegeben ist.

Der Spin des Photons kann nur die Werte annehmen m = ± 1 . Mit Blick auf die 1 + 1 1 Clebsch-Gordan-Tabelle erkennen wir, dass die nur mögliche Kopplung zweier Photonen Spins mit sich bringt j = 1 hat m = 0 . Aber auch hier hat das anfängliche Photon m = ± 1 . Daher kann der Drehimpuls nicht zusammen mit dem Viererimpuls in erhalten werden γ 2 γ .

Jedes Photon hat einen intrinsischen Drehimpuls, Spin, der nicht Null ist. Es ist 1 für ein Photon. Deshalb ist die Auswahlregel für ein Atom, das ein Photon emittiert, Delta j = +/- 1. Es liegt also eine Drehimpulsverletzung vor, wenn sich ein Photon in zwei oder mehr aufspaltet, während die Energieerhaltung noch gilt. Ein System, das ein Photon enthält, hat den Gesamtdrehimpuls 1, aber wenn sich dieses Photon in 2 Photonen aufspaltet, wird der Gesamtdrehimpuls unterschiedlich sein.

Ich bin kein Teilchenphysiker, also werde ich fragen, warum das anfängliche Photon nicht linear polarisiert werden konnte, dh in einer Überlagerung von zwei entgegengesetzten AM-Zirkularpolarisationen, so dass der erwartete Wert des beobachtbaren Spins Null war?
Eine Überlagerung ohne Drehimpuls sind ZWEI Photonen. Die beschriebene Reaktion, zwei Photonen gehen hinein und zwei verschiedene Photonen kommen heraus, wird nicht beobachtet. Wir können uns die Umkehrung einer Elektron / Positron-Vernichtung vorstellen (Photonen erzeugen Antiteilchen), aber die Geschwindigkeit einer solchen Reaktion ist WINZIG (Photonen sind groß und Elektron / Positron-Paare sind klein, sodass das Matrixelement für die Reaktion verschwinden würde, wenn diese nicht vorhanden sind Photonen werden auf einen Punkt mit Elektronenradius fokussiert).
@Whit3rd nein, das stimmt nicht, ich spreche von der Ein-Teilchen- Überlagerung der Form ( e ich ϕ | ψ + e ich θ | ψ r ) / 2 , wo ψ und ψ r sind die links- bzw. rechtshändigen Eigenzustände und ϕ , θ R .

Das sind zwei Faktoren, die in den Zerfall eines Teilchens einfließen. Da ist das Matrixelement, das aus der Grundlagenphysik kommt, und die Zustandsdichte, die aus der Kinematik kommt, hauptsächlich wie schwer das Mutterteilchen relativ zu den Tochterteilchen ist. Wenn beide sehr klein sind, wird der Zerfall verlangsamt (oder Null zu sein, verhindert, dass der Zerfall überhaupt stattfindet).

Ein Beispiel:

Das freie Neutron hat ein Matrixelement, das ungefähr dem freien Myon entspricht. Aber das Myon zerfällt in Teilchen, die viel leichter sind als es selbst (das Myon hat eine Masse von 106 MeV, und die Zerfallsprodukte zusammen haben nur eine Masse von etwa 0,5 MeV). Das Neutron hingegen ist kaum schwerer als seine Produkte: Das Neutron hat eine Masse von 939,6 MeV und seine Produkte addieren sich zu 938,8 MeV.

Das Ergebnis? Das Myon hat eine Lebensdauer von 0,0000022 Sekunden, während das Neutron eine Lebensdauer von 890 Sekunden hat. Das Neutron lebt 400 Millionen Mal länger! Im Fachjargon der Teilchenphysik nennen wir eine solche Wechselwirkung Phasenraumunterdrückung .

Eine Interaktion wie γ γ + γ wird noch stärker phasenraumunterdrückt. Tatsächlich ist der Phasenraum so unterdrückt, dass die Zerfallsrate dieser Wechselwirkung formal zu Null berechnet wird, selbst wenn das Matrixelement nicht Null ist. (Im Standardmodell ist das Matrixelement für γ 2 γ ist sowieso Null, und wir wissen aus dem Experiment, dass es trotzdem sehr klein ist)

Dasselbe Argument funktioniert für die Aufspaltung eines Photons in eine beliebige Anzahl von Photonen. Es verbietet also auch die Aufspaltung eines Photons in drei oder vier oder mehr Photonen.

Das ist dein Problem. Wir wissen nicht einmal genau, warum Teilchen zerfallen, ganz zu schweigen davon, wie viele Faktoren ein Teilchen zerfallen lässt, als ob es ein Axiom wäre. Darüber hinaus überzeugt das Einfügen konkreter Zahlen in Ihre Argumentation nur einen schwachen Verstand besser. Lassen Sie mich Ihnen etwas über dieses Fundament erzählen: Die Zeit wurde so definiert, dass sie auf dem Zerfall von Teilchen basiert. Und Sie sagen, Zerfall basiert auf Dichte und Dichte basiert auf Kinematik, die auf Bewegung basiert, die auf Entfernung und Zeit basiert . Sehen Sie jetzt den Zirkelschluss???

Es ist sicherlich thermodynamisch möglich, dass ein Photon mit hoher Energie verschwindet und eine Vielzahl von Photonen mit niedrigerer Energie erzeugt wird. Dies ist als Kaskade von Ereignissen (photoelektrische Absorption eines Photons, gefolgt von mehreren Fluoreszenzphotonen) bei der Thermalisierung eines hochenergetischen Photons, das mit Materie wechselwirkt, beobachtbar. Es ist keine einfache Photonen-in-, Zwei-Photonen-out-Reaktion, weil dies nicht als Teilchenreaktion ausgeglichen ist (Energie, Impuls und Drehimpuls nicht erhalten können).

Die übliche Kaskade, die Energie in Materie aus einem Röntgenphoton thermalisiert, könnte einige andere Photonen erzeugen, erzeugt aber hauptsächlich Phononen oder instabile Atomzustände (angeregte Elektronen). Es kann viel später Photonen geben; thermolumineszierende Geräte akkumulieren wochenlang Röntgenstrahlen, um IR-Photonen zu erzeugen, wenn sich der Strahlungsausweis im Lesegerät befindet.